Can someone help me with this math homework please!

Can Someone Help Me With This Math Homework Please!

Answers

Answer 1

Here, t represents time, which is the input of the function, so we know that 10 is the time in minutes. h(t) represents the altitude, which is the output of the function after time, t.

So, we need to substitute 10 into 210 -15t in place of t.

On simplifying 210 -15(10), we get that 60 is the altitude in feet. So, after 10 minutes, the hot air balloon is at 60 feet.
Answer 2

Hi there!  

»»————- ★ ————-««

I believe your answer is:  

[tex]h(10) \text{- the altitude of the hot air balloon after 10 minutes.}\\\\\boxed{h(10)=60}[/tex]

»»————- ★ ————-««  

Here’s why:  

⸻⸻⸻⸻

As described in the question, the function [tex]h(t)[/tex] models the total altitude of a hot air balloon over time. 't' represents the number of minutes that would be given.[tex]h(10)[/tex] would be the total altitude over 10 minutes time. To solve for the value, we would replace 't' with 10 and then evaluate.

⸻⸻⸻⸻

[tex]\boxed{\text{Evaluating the function...}}\\\\h(t) = 210 - 15t; \text{ } h(10)\\-------------\\\rightarrow h(10) = 210 - 15(10)\\\\\rightarrow h(10)=210 - 150\\\\\rightarrow \boxed{h(10) = 60}[/tex]

⸻⸻⸻⸻

»»————- ★ ————-««  

Hope this helps you. I apologize if it’s incorrect.  


Related Questions

I don’t know the answer so ignore the checked box

Answers

Answer:

CDHG

Step-by-step explanation:

Because you can go in that order and be able to get to the next letter. In the other option you cannot

What is the largest prime number less than 200?

Answers

Answer:

199

Step-by-step explanation:

A prime number (or a prime) is a natural number greater than 1 that is not a product of two smaller natural numbers.

A prime number is a number that has only two divisors: 1 and itself.

Exaples:

2, 3, 5, 7, 11, 13, 17, 19, 23 ...

We know: all primes except 2 are odd.

Let's check the number 199:

We know:

13² = 13 · 13 = 169 < 200

17² = 17 · 17 = 289 > 200

199 : 3 is not an integer

199 : 5 is not an integer

199 : 7 is not an integer

199 : 11 is not an integer

199 : 13 is not an integer

199 : 17 is not an integer

Therefore 199 is thelargest prime number less than 200.

What is the slope of the line represented by the equation y-6 = 5(x-2)?
O A. 6
O B. 5
O C. -5
O D. 2

Answers

The slope is 5. For sure

Answer:

B

Step-by-step explanation:

The equation of a line in point- slope form is

y - b = m(x - a)

where m is the slope and (a, b) a point on the line

y - 6 = 5(x - 2) ← is in point- slope form

with slope m = 5 → B

PLSSSSSSSSSSSSSSSSSSSSSSSSSSSSSSSSSSSSSSSSSSSSSSS ANSWERRRRRRRRRRRRRRRRRRRRRRRRRRRRRRRRRRRRRRRRRR

Answers

Answer:

ssssssssssssssssssssssssssssssssssssssssssssssss

Answer:

¼. 1000

Step-by-step explanation:

25% = 25/100

= 1/4

Therefore,

25% of 1,000 is the same as

1/4. 1000

Which rates are equal? Choose 2.
A. 630 miles per 9 hours
B. 1,320 miles
per 24 hours
C. 1,170 miles per 18 hours
D. 455 miles per 7 hours
E. 840 miles per 14 hours

Answers

Answer:

C. 1170 miles per 18 hours

D. 455 miles per 7 hours

Step-by-step explanation:

1170/18=65

455/7=65

Help please
If the measure of angle 6 is 140 degrees and the measure of angle 7 is (x + 30) degrees, what value of x will guarantee n ∥ m?

Answers

Answer:

x = 10

Step-by-step explanation:

If n // m , then angle 6  and angle 7 are co interior angles and they are supplementary.

∠6 + ∠7 = 180

140 + x +30 = 180

       x + 170 = 180

               x = 180 - 170

x = 10

please answer
HCF of 16 and 40

Answers

Answer:

8

Step-by-step explanation:

Hello can someone help me with this problem

Answers

no lol do it yourself

The width of this rectangle is measured as 19.4 correct to 1 decimal place. What is the lower bound for the area of the rectangle

Answers

Answer:

:D

Step-by-step explanation:

19.35 = 7062.75 mm²

please tell me the answer to this, thanks

Answers

Step-by-step explanation:

Hey there!

The equation is: 2x + 3y -12=0 (i)

From the given equation;

Slope (m) = (-coeff.of X)/(coeff.of y)

m = -2/3

Now; Let the missing component be "a".

The equation also can be written in;

The equation also can be written in;(y-8) = -2/3(x-a) (using one point formula)

3y - 24 = -2x + 2a

2x +3y -2a -24 = 0………(ii)

Since both equation are made from same coordinates, equating equation (i) and (ii).

2x + 3y -12= 2x +3y -2a -24

Cancel like terms,

-2a - 24 = -12

or, -2a = 12

or, a = -6.

Therefore, the missing value is -6.

Check:

Putting the value "a" in equation (ii).

2x + 3y -2(-6) -24 = 0

or, 2x+3y -12 = 0 (which is equal to equation (i).

Hope it helps!

Find the x-intercept of the graph of the linear equation y = −12x + 3.

Answers

Answer:

x-intercept = (1/4, 0)

Step-by-step explanation:

Just change the y to 0 if you want to find the x-intercept of a linear equation.

0=-12x+3

-12x=-3

x=1/4

y=0

x-intercept = (1/4, 0)

X= 1/4

0 = -12x + 3
X = 1/4

Find x if : (-3/7)^-19 ÷ (-3/7)^8 = (-3/7)^- 2x+1​

Answers

Answer:

x=14

Step-by-step explanation:

(-3/7) power (-19-8)= (-3/7) power -2x+1

-19-8= -2x+1

-27= -2x+1

-27-1= -2x

-28= -2x

x=28/2

x=14

PLEASE HELP I'LL MARK BRANLIEST!
Activity Gina is playing dominoes with her family. She and her brother, Greg, are playing as a team (team A) against their parents (team B). The table shows how many points the two teams have scored in each round. Gina is keeping track of the scores and wants to know which measures of center and spread best represent each team’s data.

Team A: 15 20 15 5 10 15 10 15 5 20
Team B: 20 15 25 30 20 25 0 15 25 30

Part B What is the mean score for team A? Show your calculation.

Answers

Answer:

Mean of team A is 13

Step-by-step explanation:

I hope it helps:)

If y is proportional to x, find the missing number.

A. 15
B. 12
C. 16
D. 18

HELPPP!!!!

Answers

Answer:

C. 16

Step-by-step explanation:

5 is half of 10, so 8 should be half of the next number.

Answer:

C. 16

Step-by-step explanation:

[tex] \frac{y}{x} = \frac{10}{5} = \frac{?}{8} \\ \frac{2}{1} = \frac{?}{8} [/tex]

cross multiplying

[tex]16 = ?[/tex]

so the missing number is 16

find the next four terms of the sequence 5, 4, 9, 13, 22

Answers

Answer:

38

Step-by-step explanation:

The first term of the given sequence is 9.

The second term is 13

= 9 + 4 . = 9 + 2 × 2.

The next number is 22.

= 13 + 9 . = 13 + 3 × 3 .

In the same way the next number of the given sequence will be

22 + 4 × 4 .

= 22 + 16 = 38.

How do I solve for X HELP

Answers

Answer:

x = 4 degrees

Step-by-step explanation:

The two angles have the same measurement. They are opposite angles.

26x + 7 = 31x - 13

Group like terms:

26x - 31x = -13 - 7

-5x = -20

x = 4

Check if you want to make sure.

Step-by-step explanation:

[tex]here \: is \: your \: solution : - \\ \\ both \: angles \: are \: equal \: (vertical \: anges) \\ \\ 26x + 7 = 31x - 13 \\ \\ solving \: the \: equation \: \\ \\ 26x - 31x = - 13 - 7 \\ \\ - 5x = 20 \\ \\ x = -20 \div 5 \\ \\ x = \: 4 \: degrees \: \: \: \:(Answer ✓✓✓( \\ \\ \huge\mathcal\blue{happy \: life (. ❛ ᴗ ❛.) }[/tex]

A college with a graduating class of 4000 students in the year 2010 predicts that its graduating class will grow 5% per year.
Using an exponential function to model the number of students y in the graduating class t years after 2010 to predict the number
of students in 2017?

Answers

Hello,

[tex]u_0=4000\\u_1=4000*1.05 (for\ year\ 2011)\\\\u_n=4000*1.05^n\\So:\\year\ 2017: u_7=4000*1.05^7=5628,401690625\ \approx{5628}[/tex]

Using an exponential function, the number of students y in the graduating class 7 years after 2010 i.e. in 2017 will be 5628.4

What is an exponential function?

y = abˣ, where a is the initial population, b is the rate, and x is the time, is the standard exponential function.

How to solve this problem?

Here initial student population = 4000.

Rate = 5% = (100 + 5)/100 = 1.05

Time = 7 years.

Now, in 2017, the population will be y = 4000 * (1.05)⁷ = 5628.401691 ≅ 5628.4.

Therefore, using an exponential function, the number of students y in the graduating class 7 years after 2010 i.e. in 2017 will be 5628.4

Learn more about exponential function here -

https://brainly.com/question/14551308

#SPJ2

what s the equation of the line through the origin and (-2, 3)

Answers

Answer:

y=[tex]\frac{-3x}{2}[/tex]

Step-by-step explanation:

Hi there!

We need to find the equation of the line that passes through the origin (the point (0,0)) and (-2,3)

There are 3 ways to write the equation of the line, although the most common way is slope-intercept form.

Slope-intercept form is given as y=mx+b, where m is the slope and b is the y intercept

So we need to find the slope of the line first

The formula for the slope (m) calculated from two points [tex]\frac{y_{2}-y_{1}}{x_{2}-x_{1}}[/tex] where ([tex]x_{1}[/tex],[tex]y_{1}[/tex]) and ([tex]x_{2}[/tex], [tex]y_{2}[/tex]) are points

We have the needed information to calculate the slope, but let's label the values of the points to avoid any confusion

[tex]x_{1}[/tex]=0

[tex]y_{1}[/tex]=0

[tex]x_{2}[/tex]=-2

[tex]y_{2}[/tex]=3

Now substitute their values into the equation and find m

m=[tex]\frac{y_{2}-y_{1}}{x_{2}-x_{1}}[/tex]

m=[tex]\frac{3-0}{-2-0}[/tex]

subtract

m=[tex]\frac{3}{-2}[/tex]

so the slope of the line is [tex]\frac{3}{-2}[/tex]. It can also be rewritten as [tex]\frac{-3}{2}[/tex]

Here is the equation of the line so far:

y=[tex]\frac{-3x}{2}[/tex]+b

we need to find b

As the equation passes through both (0,0) and (-2,3), we can use either one of them to solve for b

Let's take (0,0) for this case

Substitute 0 as x and 0 as y

0=-[tex]\frac{3}{2}[/tex](0)+b

multiply

0=0+b

add 0 to both sides

0=b

So b is 0

The equation of the line therefore is y=[tex]\frac{-3x}{2}[/tex]

Hope this helps!

Drag the tiles to the correct location on the image.Not all tiles will be used.​

Answers

Answer:

Given,

side=4√5 millimetres

=4×4√5

=16√5

Perimeter= 16√5 milimetre

Area= side× side

=4√5 × 4√5

=80

Area= 80 square milimeters

------------------------------

hope it helps...

have a great day!

Answer:

solution:-

Given,

side=4√5 mm

=4×4√5

=16√5

Perimeter= 16√5 mm

Area= side× side

=4√5 × 4√5

=80

Area= 80 square milimeters

------------------------------

hope it is helpful to you

stay safe healthy and happy

Hello, Brainly community!

This question is for all of those Calculus people out there.

The volume of a swimming pool is changing with respect to time, such that the volume is given by W(t), where W(t) is measured in cubic centimeters and t is measured in seconds. A tangent line is shown for W(t) at t = 3 seconds. Determine the best estimate for the value of the instantaneous rate of change of W(t) when t = 3.
(I've narrowed down the answer choices to 2, and just really need to find the right way of thinking to find the answer)

(A) W(lim t) as t goes to 3.
(B) [W(3.1) - W(2.9)] / 0.2.

Thank you in advance!

Answers

Answer:

(B)  [tex]\displaystyle \frac{W(3.1) - W(2.9)}{0.2}[/tex]

General Formulas and Concepts:

Calculus

Limits

Derivatives

The definition of a derivative is the slope of the tangent line.

Derivative Notation

Instantaneous Rates

Tangent Line: [tex]\displaystyle f'(x) = \frac{f(b) - f(a)}{b - a}[/tex]

Step-by-step explanation:

Since we are trying to find a rate at which W(t) changes, we must find the derivative at t = 3.

We are given 2 close answer choices that would have the same numerical answer but different meanings:

(A)  [tex]\displaystyle \lim_{t \to 3} W(t)[/tex](B)  [tex]\displaystyle \frac{W(3.1) - W(2.9)}{0.2}[/tex]

If we look at answer choice (A), we see that our units would simply just be volume. It would not have the units of a rate of change. Yes, it may be the closest numerically correct answer, but it does not tell us the rate at which the volume would be changing and it is not a derivative.

If we look at answer choice (B), we see that our units would be cm³/s, and that is most certainly a rate of change. Answer choice (B) is also a derivative at t = 3, and a derivative tells us what rate something is changing.

∴ Answer choice (B) will give us the best estimate for the value of the instantaneous rate of change of W(t) when t = 3.

Topic: AP Calculus AB/BC (Calculus I/I + II)

Unit: Differentiation

Book: College Calculus 10e

A soccer team made $575.75 from selling popcorn at a concession stand during a tournament. The popcom cost the
team $65.00. What is the amount of money earned for each player if the profit is divided equally among the 15
players? Round to the nearest cent.
O S34.05
0 $42.72
57.661.25
59.611.25 hurry I'm being timed!

Answers

Answer:

$34.05

Step-by-step explanation:

can anyone answer this? (2y²-3xy)÷y.​

Answers

the answer would be y x(2y-3x)

Answer:

2y - 3x

Step-by-step explanation:

Given

(2y² - 3xy) ÷ y

Divide each term on the numerator by y , that is

[tex]\frac{2y^2}{y}[/tex] - [tex]\frac{3xy}{y}[/tex] ( cancel y on numerator/ denominator of both fractions )

= 2y - 3x

A 4-column table with 4 rows. Column 1 is labeled number of friends with entries 3, 5, 7, 9. Column 2 is labeled Carnival Cost with entries 51.5, 75.5, 99.5, 123.5. Column 3 is labeled Aquarium Cost with entries 43.5, 72.5, 101.5, 130.5. Column 4 is labeled Wave Pool Cost with entries 50.25, 61.25, 85.75, 110.25.
Gale wants to compare the cost of the events.

Aquarium: $14.50 each ticket

Carnival: c = 15.5 + 12f

Wave pool: $16.75 each, but $12.25 each for groups larger than 4

Which of the claims Gale makes is true?
The table represents all possibilities.
The carnival always costs the least.
The Aquarium always costs the most.
(7, 101.5) is an ordered pair used to represent the aquarium cost.

Answers

Answer:

D.(7, 101.5) is an ordered pair used to represent the aquarium cost.

Hope it helps :)

Step-by-step explanation:

Answer: it is d - (7, 101.5) is an ordered pair used to represent the aquarium cost

Step-by-step explanation: got it right on edge 2022

Find the measure of the indicated angle.

Answers

2x + 40 = 180

2x = 140

x= 70

answer is 70


PLEASE HELP.
if v1….

Answers

I believe the answer is 4.4

PLEASE HELP!!!!!!!!!





A car manufacturer does performance tests on its cars. During one test, a car starts from rest, and accelerates at a constant rate for 20 seconds. Another car starts from rest three seconds later, and accelerates at a faster constant rate. The equation that models the distance (d) in metres the first car travel travels is: d=1.16t^2, where t is time, in seconds, after the car starts. The equation for the second car is: d= 1.74(t-3)^2


a. In the context, what is a suitable domain for the graph of this system?


b. At what time will both cars have driven the same distance?


c. How far will they have driven at this time?

Answers

Answer:

15.66

Step-by-step explanation:

Which equation in slope-intercept form represents a line that is parallel to y=-4x-5 and passes through the points (0,0)?

Answers

Answer:

y = 0x

Step-by-step explanation:

y=-4x-b

0 = -4(0) + b

0 = 0 + b

0

PLEASE HELP NEED HELP W ALGEBRA 2 QUESTION!!

the graph of f(x) = 3 sqr root x+8 is shown which statement is true

1. The function is only increasing x > -8
2. The function is only increasing when x > 0
3. The function is always decreasing
4. The function is always increasing

Pls say what number choice in your answer! Thanks!

Answers

Answer:

2. increasing when x > 0

the result of root is a positive number

and since there is no value of neg number in roots so it's increasing when x>0

plus in zero it self has a positive number (0,8)

but u cannot find either it's increasing or decreasing in zero bcoz in first derivative test

when x =0

y= not defined

A company uses two vans to transport
workers from a free parking lot to the
workplace between 7:00 and 9:00 a.m.
One van has 6 more seats than the other.
The smaller van makes two trips every
morning while the larger one makes only
one trip. The two vans can transport 57
people, maximum.
How many seats does the larger van have?

Answers

x is the seats in the small van.

y is the seats in the large van = (x + 6). Since the large has 6 more seats.

Since the 2 vans can transport 57 people.

Small van makes 2 trips = 2x.

Large van makes one trip = (x + 6).

2x + (x +6) = 57

2x + x + 6 = 57

3x + 6 = 57

3x = 57 -6

3x = 51

x = 51/3

x = 17.

The larger van has (x + 6) = (17 + 6) = 23.

The larger van has 23 seats.

Answer:

The larger van has 23 seats

Step-by-step explanation:

Create a system of Equations:

1. Define variables.

> let x=small van and y=larger van

2. create 2 equations based on the information given.

> y = 6 + x

> 2x + y = 57

3. Use any method to solve

Substitution: 2x + (6 + x) = 57. x=17

                      now plug x in to the original equation to solve for y (the larger van)

                      y = 6 + 17 and y = 23

Elimination: 2y = 12 + 2x

                     y = 57 - 2x

         3y = 69  and y = 23

(NEED THIS ASAP)

Tests show that the hydrogen ion concentration of a sample of apple juice is 0.0003 and that of ammonia is 1.3 x 10-9. Find the approximate pH of each liquid using the formula pH = -log (H+), where (H+) is the hydrogen ion concentration The pH value of the apple juice is___ The pH value of ammonia is____

1.pH of apple juice
A. 8.11
B. 1.75
C. 3.5
D. 2.1

2. pH of ammonia
A. 1.1
B. 7.0
C. 5.4
D. 8.9​

Answers

Answer: I believe but not 100% sure

1) C

2) B

Step-by-step explanation:

The pH value of the apple juice is 3.5, option C) is the correct answer.

The pH value of the ammonia is 8.9, option D) is the correct answer.

What is pH of solution?

The pH of a solution is defined as the logarithm of the reciprocal of the hydrogen ion concentration  [H+] of the given solution.

From the formular;

pH = -log[ H⁺ ]

Given the data in the question.

For the Apple juice;

hydrogen ion concentration H⁺ = 0.0003 pH of the apple juice pH = ?

pH = -log[ H⁺ ]

pH = -log[ 0.0003 ]

pH = 3.5

The pH value of the apple juice is 3.5

Option C) is the correct answer.

For the ammonia;

hydrogen ion concentration H⁺ = 1.3 × 10⁻⁹pH of the ammonia pH = ?

pH = -log[ H⁺ ]

pH = -log[ 1.3 × 10⁻⁹]

pH = 8.9

The pH value of the ammonia is 8.9

Option D) is the correct answer.

Learn more about pH & pOH here: brainly.com/question/17144456

#SPJ2

Other Questions
It is permissible to use a directional H1 when there are good theoretical as well as strong supporting data to justify the predicted direction.a. trueb. false Select all of the following statments that are true Given : AB = BC and BC = CD, AB = 3x - 1 and CD = 2x + 3 Prove: BC = 11 plz help me what was George Whitefield preaching style during the Great Awakening? Lee is evaluating an informational text about fire safety to determine whether it addresses campfires. Which two print features will answer his questions most quickly?indextitlecaptionsheadingstable of contents An analysis of 99 Wall Street traders showed that 32 of their stock picks beat the market average. What is the estimate of the population proportion Two objects attract each other with a gravitational force of magnitude 1.00 3 1028 N when separated by 20.0 cm. If the total mass of the two objects is 5.00 kg, what is the mass of each A major component of gasoline is octane when octane is burned in air it chemically reacts with oxygen to produce carbon dioxide and water what mass of carbon dioxide is produced by the reaction of oxygen Fred and Wayne are 4-year-olds. When they are together, they often wrestle, run, race, push, and shove each other. Although their activities often aggravate their parents, these activities will what organelles are not found in plant cells? Which is equivalent to 2^5? The radius of a sphere is increasing at a rate of 2 mm/s. How fast is the volume increasing (in mm3/s) when the diameter is 60 mm? Manuel delega las responsabilidades a los voluntarios en el hogar de ancianos.Which pronoun would you use to replace the indirect object in the sentence above? (1 point)Select one:a. Lab. Loc. Lesd. Los In marketing, we use which terms? (needs, wants, or demands). Why? World-Tour Co. has just now paid a dividend of $2.83 per share (Div0); its dividends are expected to grow at a constant rate of 6% per year forever. If the required rate of return on the stock is 16%, what is the current value of the stock after paying the dividend A number of computers have been reported to be getting incorrect IP addresses. After doing some investigation, an unauthorized DHCP server has been determined to be the cause. Which step in the troubleshooting process should be performed next If an insurance policy covers individual losses up to $2000 and has a deductible of $200, the insured will be paid how much in the event of a covered loss of $2500A. $2,300B. $2,000C. $1,800D. $2,500 Identify every authority cited in the opinion (note that some sources may be cited more than once) and then do the following: List all primary mandatory authority. List all primary persuasive authority.List all secondary persuasive authority. In the history of Major League Baseball, the most valuable player award has been won by players from 7 different countries. which countries were they How did bombing change thepicture of war in WWII? simple answer please